Nouvelle démonstration du grand théorème de Fermat - Page 4
Discussion fermée
Page 4 sur 8 PremièrePremière 4 DernièreDernière
Affichage des résultats 91 à 120 sur 237

Nouvelle démonstration du grand théorème de Fermat



  1. #91
    invite636fa06b

    Re : Nouvelle démonstration du grand théorème de Fermat


    ------

    Citation Envoyé par rvz Voir le message
    Ok, je vais la poster.

    Premier bout : Les triplets pythagoriciens primitifs sont de la forme (n^2-1, 2n,n^2+1). (S'inspirer d'une paramétrisation rationnelle du cercle unité)
    __
    rvz
    Non Les triplets pythagoriciens primitifs sont de la forme (n^2-p^2, 2np,n^2+p^2).
    en faisant p=1 tu ne couvres pas tous les cas de triplets premiers entre eux

    -----

  2. #92
    Gaétan Mbama

    Re : Nouvelle démonstration du grand théorème de Fermat

    Salut les gars!

    Admettons que x, y et z ne sont pas les côtés d'un triangle rectangle.

    Considérons l'équation de Fermat x^n+y^n=z^n

    pour n=2 nous avons x²+y²=z²
    n'est ce pas l'équation de Pythagore? A moins que vous me dites le contraire!

    Si c'est le cas, que dirions nous alors à propos de x, y
    et z ?

  3. #93
    leg

    Re : Nouvelle démonstration du grand théorème de Fermat

    Citation Envoyé par skydancer Voir le message
    Et je pense que c'est de quoi Geatan parle en considerant des x,y,z cotés d'un triangle rectangle.
    si tu suppose que x,y et z sont des produits de puissance N par ex N = 3, tu est d'accord que nous avons trois entiers positifs
    (x3);(y3), et (z3)
    est que ces trois entier peuvent vérifier l'équation de Fermat par exemple pour N = 6 ; pour le savoir il me suffirait:
    a) de mettre ces trois entiers au carré

    b) d'élever x, y et z à la puissance 6

    c) d'élever les trois carrés A, B et C correspondant à
    cette égalité; à la puissance N = 3

    je ne peux faire un triangle rectangle qu'avec les entiers de la condition a) et si cette condition me permet de tracer effectivement un triangle rectangle alors j'ai une solution de l'équation de Fermat que je peux vérifier, avec les trois conditions a) ,b) et c)

    la racine carré des entiers élevés a la puissance N=6 de la condition b) sont les entiers de la condition a) mais la racine carrée des entiers carré, élevé à la puissance N = 3, de la condition c) sont aussi ceux de la condition a)

    par exmp, x=2
    condition a) x=8 =23=x3
    condition c) A=4 , A3=64 et racine carré de 64 = x3=8
    (" et effectivement comme tu l'a dit, une solution dans N=6 serait aussi soltuion dans N = 2 ; toutes solutions dans N pair > 2 et forcement solution dans cette dernière puissance N=2 ")
    Alors je peux faire un triangle rectangle et verifier l'équation de Fermat.

    or ceci dit, une solution dans N = 6 et aussi solution dans N = 3 donc il ne sert a rient de démontrer uniquement N = 6
    si je ne veux qu'une puissance N impair et première, et verifier l'équation de Fermat qui pourrait exister uniquement dans cette puissance N, par ex N=3 avec une relation de pythagore, il me faut montrer que je peux faire un triangle rectangle avec les racines carrées de x, y et z élevé à la puissance N = 3, et il est évident que ces trois racines carrées ne sont des entiers, pour la bonne et simple raison que ce serait des cubes donc solution dans N = 6 ce que je ne veux pas dans ma supposition.
    Ou alors montrer que je ne peux pas faire de triangle rectangle avec ces racines carrées ce qui revient au même; et ce quelque soit la puissance N>2

  4. #94
    invite6b1e2c2e

    Re : Nouvelle démonstration du grand théorème de Fermat

    Citation Envoyé par zinia Voir le message
    Non Les triplets pythagoriciens primitifs sont de la forme (n^2-p^2, 2np,n^2+p^2).
    en faisant p=1 tu ne couvres pas tous les cas de triplets premiers entre eux

    Ah ! Je me disais bien que c'était trop facile !

    Bon, alors après, faut jouer un peu avec les coefficients n et p. (On peut se ramener à n et p premiers entre eux.)

    Bonne chance,
    __
    rvz

  5. #95
    leg

    Re : Nouvelle démonstration du grand théorème de Fermat

    Citation Envoyé par rvz Voir le message
    Ok, je vais la poster.

    Donc on peut écrire x^2 = n^2 -1,
    y^2 = 2n, Z = n^2 +1 ...
    Donc (n-x)(n+x) = 1. .

    En fait, ça me paraît un peu bizarre que ça marche comme ça. J'ai du me planter quelque part, à vous d'être critique
    rvz
    Z = 13, = N² + u² où N =3 et u =2
    donc si Z = n² + 1 = 13..??? tu n'a que 4 et 9.
    comment tu fais? pour trouver toutes les solutions de la formule pythagorique, que Zinia vient de te montrer...et par conséquent tu ne pourrait démontrer toutes les solution qui pourrait exister dans N = 4
    A+

  6. #96
    leg

    Re : Nouvelle démonstration du grand théorème de Fermat

    Citation Envoyé par rvz Voir le message
    Ah ! Je me disais bien que c'était trop facile !

    Bon, alors après, faut jouer un peu avec les coefficients n et p. (On peut se ramener à n et p premiers entre eux.)

    Bonne chance,
    __
    rvz
    tu peux le voir sous cet angle supposons X carré (x²)
    (x²) = (n+p)(n-p) le produit de ces nombre premiers entr eux, étant carré, ces nombre sont eux même des carrés ainsi que n et p...
    à toi la suite, il suffit de montrer que n et p sont carré et mois je conclu...
    A+ dans une petite heure.

  7. #97
    invite6b1e2c2e

    Re : Nouvelle démonstration du grand théorème de Fermat

    u = n+p et v = n-p ne sont pas premiers entre eux à priori, mais leur pgcd divise 2.

    Si u et v sont pairs, alors leur pgcd est 2, donc x^2 est pair. Mais la forme de y^2 donne clairement que y^2 est pair. Impossible car alors le triplet ne serait pas primitif. On peut diviser par 2 !

    Si il y en a exactement un seul parmi u et v est impair, et pgcd(u,v) = 1.
    Mais 2n = u +v est impair donc j'ai une contradiction.

    Donc u et v sont impairs et pgcd(u,v) = 1. Donc je peux les écrire sous forme de carrés, u = a^2, v = b^2 avec a et b impair et premiers entre eux.

    Du coup, l'équation donne

    a^2 b^2 + (a^4 - b^4)/2 = (a^4 + b^4)/2
    Donc a^2 b^2 = b^4, ce qui est contradictoire avec la condition pgcd(a,b) = 1, sauf si a ou b =1, ce qui doit donner une solution triviale, car cette dernière équation donne au choix b^2 = 0 ou 1, a^2 =1 pour compléter le système.

    J'ai encore craqué ?
    __
    rvz

  8. #98
    invite79d10163

    Re : Nouvelle démonstration du grand théorème de Fermat

    Citation Envoyé par zinia Voir le message
    Bonsoir skidancer et leg

    Vous avez l'air bien partis pour un débat sans fin et je n'a pas l'intention ni la prétention de vous départager.
    Simplement je voulais clarifier un point :
    on peut effectivement prouver qu'un triplet de nombres réels positifs x,y,z ne peut pas vérifier deux relations :

    La démonstration élémentaire utilise le fait que la fonction est concave lorsque a>1.
    Il n'est pas nécessaire de poser des conditions d'appartenance à N pour les variables ou les exposants, simplement il faut éliminer les solutions triviales (avec des 0 et des 1) et prendre des exposants positifs
    Ok c'est bien ce que je me disais...

    donc finalement un triplet pythagoricien (n=2) ne peut pas etre un triplet de fermat ( n>2 ).
    Sans rancunes

  9. #99
    leg

    Re : Nouvelle démonstration du grand théorème de Fermat

    Citation Envoyé par Gaétan Mbama Voir le message
    Salut les gars!

    Admettons que x, y et z ne sont pas les côtés d'un triangle rectangle.

    Considérons l'équation de Fermat x^n+y^n=z^n

    pour n=2 nous avons x²+y²=z²
    n'est ce pas l'équation de Pythagore? A moins que vous me dites le contraire!

    Si c'est le cas, que dirions nous alors à propos de x, y
    et z ?
    que c'est un triplet pythagoricien, qui verifie l'équation de Fermat pour N =2 et de plus: x,y et z peuvent être les côtés d'un triangle rectangle. que ce sont trois entiers non nul, qui peuvent être paramettré avec la formule des triplets pythagoriciens..

  10. #100
    leg

    Re : Nouvelle démonstration du grand théorème de Fermat

    Citation Envoyé par rvz Voir le message
    u = n+p et v = n-p ne sont pas premiers entre eux à priori,

    Donc u et v sont impairs et pgcd(u,v) = 1. Donc je peux les écrire sous forme de carrés, u = a^2, v = b^2 avec a et b impair et premiers entre eux.

    Du coup, l'équation donne

    a^2 b^2 + (a^4 - b^4)/2 = (a^4 + b^4)/2
    Donc a^2 b^2 = b^4, ce qui est contradictoire avec la condition pgcd(a,b) = 1, sauf si a ou b =1, ce qui doit donner une solution triviale, car cette dernière équation donne au choix b^2 = 0 ou 1, a^2 =1 pour compléter le système.

    J'ai encore craqué ?
    __
    rvz
    n et p, sont les paramettres des triplets Pyth
    j'ai donc supposé que X est un carré soit (n+p)(n-p) =
    (u v) = a² b² = n² - p²
    n et p sont de parité différente.
    u et v ainsi que X et Z sont donc impair.

    mais j'ai aussi Z qui est un carré (z²) qu'est ce que l'on peut dire sur n et p..?
    peuvent ils être deux carrés de sorte que dans ta supposition puisque u et v peuvent s'ecrire sous forme de deux carrés a² et b² ;
    tel que n² + p² = a² = u et n²-p² = b² = v....?

  11. #101
    leg

    Re : Nouvelle démonstration du grand théorème de Fermat

    bonjour rvz
    je ne vois pas ou tu veux en venir...
    tu dis:
    rvz
    u = n+p et v = n-p ne sont pas premiers entre eux à priori,:
    a) pouquoi puisque en dessous tu dis que (u,v) le sont
    rvz
    Donc u et v sont impairs et pgcd(u,v) = 1. Donc je peux les écrire sous forme de carrés, u = a^2, v = b^2 avec a et b impair et premiers entre eux

    b) si tu peux les écrire sous forme de carrés, alors:
    X = n² - p² =(u v) est un carré (x²) impair ok
    rvz
    Du coup, l'équation donne

    a^2 b^2 + (a^4 - b^4)/2 = (a^4 + b^4)/2
    Donc a^2 b^2 = b^4, ce qui est contradictoire avec la condition pgcd(a,b) = 1, sauf

    c)(a^4 - b^4)/2 je comprend que cela correspondrai a Y = 2np
    car par ex a²=9 et b²=1 donc (x²) = 9 et n =5 , p=4 donc 2np=40 = (a^4 - b^4)/2. ok
    mais apres si tu veux montrer que a^2 b^2= (x²)²
    alors (x²)² = (n²+p² + 2np) (n²-p² - 2np)
    mais pour quoi faire ?

  12. #102
    leg

    Re : Nouvelle démonstration du grand théorème de Fermat

    voila ce que j'ai trouvé rvz.

    Lemme 4 : Si un produit de nombres premiers entre eux (deux à deux) est un carré, alors ces nombres sont eux-mêmes des carrés.
    Preuve : Supposons pgcd(p,q,r) = 1 (pris 2 à 2, le pgcd est 1) et pqr = t^2. L'entier t se décompose en un produit de facteurs premiers. Par suite t^2 est un produit de facteurs premiers dont les exposants sont pairs. Puisque pqr = t^2, les diviseurs premiers de p, q et r sont extraits de ceux de t. Soit d un diviseur premier de t, apparaissant, dans t^2, sous la forme d2n. Il ne peut diviser deux des nombres p, q et r. S'il divise p, alors d2n qui divise t2 divise nécessairement p (théorème de Gauss). Donc, par épuisement des cas, p est un carré et on obtiendra que q et r sont des carrés.
    Les lemmes 2 et 4 permet d'affirmer que p, q et p2 - q2 sont des carrés. Posons p = m2 , q = n2 et p2 - q2 = r2. Ainsi :
    r2 = p2 - q2 = m4 - n4 = (m2 + n2)(m2 - n2) ;
    .............................. .............................. ................
    on peut pour la comodité de la suite de ton raisonnement remplacer les paramètres p et q ci dessus par n et p et r² par (x²)
    A toi..

  13. #103
    invite6b1e2c2e

    Re : Nouvelle démonstration du grand théorème de Fermat

    Salut leg,

    Je suis désolé, j'ai pas vraiment le temps en ce moment d'y réfléchir posément. Donc je t'apporte une réponse rapide et certainement encore incomplète.

    1/ Je dis au début que à priori, pgcd(u,v) peut valoir 1 ou 2. Par une étude de cas, je montre que pgcd(u,v) = 2 amène une contradiction.

    2/ Donc pgcd(u,v) = 1.
    Après, comme uv est un carré, alors u et v sont des carrés. C'est une conséquence du lemme que tu as démontré, que j'avais supposé connu. Pour t'en convaincre en 10 secondes, décomposer en produit de facteurs premiers est redoutable.

    3/ On bidouille encore un peu l'équation et on arrive à une contradiction. C'est sur cette partie là que j'ai évidemment craqué. Mais je suis sûr qu'on peut y arriver par là, quitte à travailler modulo 2, 4 ou 8, modulo ou les carrés des impairs ont le bon gout d'être toujours égales à 1...

    C'est le plan de ce que j'ai proposé la dernière fois.
    Si tu en as marre de chercher, je répéte que c'est fait dans les toutes premières pages du Samuel de théorie des nombres (qui au passage est l'un des livres les plus clairs que j'ai vu sur le sujet). Si tu ne peux pas te le procurer, je te donnerai la preuve quand je l'aurai sous la main, c'est à dire pas avant une bonne quinzaine de jours.
    __
    rvz, qui n'arrête pas d'écrire n'importe quoi en ce moment, désolé

  14. #104
    invite636fa06b

    Re : Nouvelle démonstration du grand théorème de Fermat

    Citation Envoyé par rvz Voir le message
    3/ On bidouille encore un peu l'équation et on arrive à une contradiction. C'est sur cette partie là que j'ai évidemment craqué. Mais je suis sûr qu'on peut y arriver par là, quitte à travailler modulo 2, 4 ou 8, modulo ou les carrés des impairs ont le bon gout d'être toujours égales à 1...
    __
    rvz, qui n'arrête pas d'écrire n'importe quoi en ce moment, désolé
    Bonjour,
    Je ne comprends pas, ta preuve est un peu elliptique mais elle tient la route de a (ou r) à z.

  15. #105
    invite6b1e2c2e

    Re : Nouvelle démonstration du grand théorème de Fermat

    Citation Envoyé par zinia Voir le message
    Bonjour,
    Je ne comprends pas, ta preuve est un peu elliptique mais elle tient la route de a (ou r) à z.
    Cool
    Je pensais qu'il y avait des fautes, vu que leg semblait sceptique. Comme je n'ai pas pris la peine de vraiment vérifier, je me suis dit qu'il devait y en avoir. D'autant plus qu'en ce moment, mon directeur de thèse n'arrête pas de trouver des erreurs partout dans ce que je fais

    __
    rvz, qui devrait être en train de réfléchir à des problèmes de bases de Riesz et de "frame" dans des espaces fonctionnels

  16. #106
    Gaétan Mbama

    Re : Nouvelle démonstration du grand théorème de Fermat

    Martini salut !

    Je réponds à tes pertinentes observations.

    1°) Je suis d’accord avec toi quand tu affirmes que lorsqu’on suppose que x, y et z sont les mesures (entières) des côtés d’un triangle rectangle, alors on résout le système de deux équations x²+y²=z² et xn+yn=zn.

    2°) Je suis encore d’accord avec toi quand tu dis que ce système n’est pas le GTF. Moi, je ne l’ai pas dis non plus dans mon texte.

    3°) J’ai plutôt dis ceci : on part de ce système (page 4, Titre 3.3, formule (c)) pour aboutir au GTF (page 3, Titre 2.2, formule (b)).

    4°) En supposant x, y et y comme mesurant les côtés d’un triangle rectangle d’hypoténuse z, j’exécute une manœuvre conceptuelle de changement de cadre mathématique qui me permet d’approcher le problème géométriquement. Cette manœuvre, en soi, n’altère pas le contenu de l’énoncé du GTF. ( Salut Zinia, tu as compris !)

    5°) Tu dis que cette hypothèse est restrictive. Mais je ne vois pas quoi elle est restrictive. Je vois seulement qu’elle me permet d’écarter les solutions ‘’triviales’’ (0,0,0), (0,1,1), (1,0,1) et (1,1,0) ( comme l’a souligné à juste titre rvz). rvz, tu t’es trompé, ce sont des solutions « triviales » et non « non triviales » comme tu le dis dans ton post 21.

    6°) Pourquoi voulez-vous que x, y et z soient forcément des nombres entiers.
    Relisons ensemble le GTF tel que traduit du latin (Martini, tu veux des références?):

    « Il n’est pas possible de décomposer un cube en somme de deux cubes, une puissance quatrième en somme de deux puissances quatrièmes et généralement aucune puissance d’exposant supérieur à deux en deux puissances de même exposant ».

    Dites-moi, dans cet énoncé, où est ce qu’il est dit que x, y et z doivent être exclusivement des entiers ? Il peut aussi s’agir de rationnels!

    Question: en quoi, par exemple, 33 est ‘’plus cube’’ que 3,53 ?


    JE REPONDS MAINTENANT A SKYDANCER

    Leg a éloquemment répondu à tes questions. Mais je vais ajouter ceci :

    1°) D’abord, je ne sais pas ce que tu entends par « trivial ». A mon sens, trivial est synonyme de « évident » c'est-à-dire ce qui saute aux yeux tout de suite.

    2°) Je ne te comprend pas lorsque tu dis qu’il est trivial que x3+y3=z3 soit impossible quand on pose x²+y²=z². Ce fait n’est aucunement trivial (comprenez, évident) puisque nous le savons tous de Fermat qui l’avait conjecturé.

    La théorie des nombres, ce n’est pas trop mon truc (leg, tu ne m’invites pas au resto ?). Mais je crois comprendre, skydancer, que ton face à face avec leg t’a fait prendre conscience que ton truc trivial avec des n pairs et/ou impairs n’était pas cohérent.

    Question : un triplet d’entiers non nuls (x,y,z) peut-il vérifier simultanément l’égalité de Fermat dans deux exposants différents ? (rvz, j’ai deviné, ne t’accroches pas aux solutions triviales, s’il te plait !).

    NB : ne pas confondre l’égalité de Fermat (xn+yn=zn) du GTF ( quand apparaît la condition n>2).

    Réponse : D’après le GTF, on ne peut considérer que les exposants qui sont inférieurs ou égaux à deux c'est-à-dire 1 et 2 par ce que les exposants n>2, l’égalité de Fermat est impossible.

    Sommes nous d’accord ?

    Ainsi, la question qui est posée revient donc à celle de savoir si un triplet (x,y,z) d’entiers non nuls peut vérifier simultanément x+y=z et x²+y²=z².

    La réponse est évidemment non, puisque cela nous amène à une absurdité. Vous savez pourquoi ? (Evariste_galois, skydancer, rvz et Martini – répondez !)

    3°) Ce qui est important pour moi, c’est d’expliquer pourquoi cela doit se passer ainsi c'est-à-dire, pourquoi dans un triangle rectangle l’égalité x²+y²=z² est résoluble et que x3+y3=z3 ne l’est pas et d’une manière générale pourquoi l’équation xn+yn=zn est irrésoluble quand n>2.

    L’approche que je propose permet d’expliquer cela.

    Leg salut !

    A ta question, je répond par oui, bien sûr les côtés x, y et z sont des entiers non nuls y compris leurs racines carrées puisque nous sommes dans le cadre du GTF.

    Par contre, quand j’écris par exemple l’égalité x1,15+y1,15=z1,15 , pour moi, elle a un sens géométrique précis c'est-à-dire x, y et z représentent les côtés d’un triangle obtus dont je peux, avec précision, déterminer l’angle. Donc, je peux passer d’un exposant n à un autre par une simple rotation. Ceci explique pourquoi un même triplet ne pas vérifier l’égalité de Fermat dans deux exposant différents.

    La méthode permet d’établir un lien entre l’exposant n de l’égalité de Fermat xn+yn=zn et l’angle entre les cathètes du triangle. (La formule n’est pas dans le texte).

    En fait, le théorème de Fermat apparaît comme une singularité propre aux triangles rectangles.

    Question ?

    Amicalement !

  17. #107
    invite79d10163

    Re : Nouvelle démonstration du grand théorème de Fermat

    Citation Envoyé par Gaétan Mbama Voir le message
    Martini salut !

    6°) Pourquoi voulez-vous que x, y et z soient forcément des nombres entiers.
    Relisons ensemble le GTF tel que traduit du latin (Martini, tu veux des références?):

    « Il n’est pas possible de décomposer un cube en somme de deux cubes, une puissance quatrième en somme de deux puissances quatrièmes et généralement aucune puissance d’exposant supérieur à deux en deux puissances de même exposant ».

    Dites-moi, dans cet énoncé, où est ce qu’il est dit que x, y et z doivent être exclusivement des entiers ? Il peut aussi s’agir de rationnels!
    Le probleme, c'est que la décomposition est tout a fait possible pour des nombre réels ou p-adique.... Ce n'est que pour des entiers que ca ne marche pas !


    Citation Envoyé par Gaétan Mbama Voir le message

    JE REPONDS MAINTENANT A SKYDANCER

    2°) Je ne te comprend pas lorsque tu dis qu’il est trivial que x3+y3=z3 soit impossible quand on pose x²+y²=z². Ce fait n’est aucunement trivial (comprenez, évident) puisque nous le savons tous de Fermat qui l’avait conjecturé.
    ben si x²+y²=z² alors z^3 = (x^2 + y^2)^(3/2)
    d"apres la formule du binome (x^2 + y^2)^(3/2) \= x^3 + y^3
    Voila c'est ce que j'appelle trivial car il faut une ligne pour le monter.


    Citation Envoyé par Gaétan Mbama Voir le message
    La théorie des nombres, ce n’est pas trop mon truc (leg, tu ne m’invites pas au resto ?). Mais je crois comprendre, skydancer, que ton face à face avec leg t’a fait prendre conscience que ton truc trivial avec des n pairs et/ou impairs n’était pas cohérent.
    Je suis pas d'accord du tout. Réflechis et tu verras que x,y,z ne peut pas etre solution de l'equation de fermat simultanement pour n=2 et n>2 et c'est simple à montrer avec la formule du binome.


    Citation Envoyé par Gaétan Mbama Voir le message
    NB : ne pas confondre l’égalité de Fermat (xn+yn=zn) du GTF ( quand apparaît la condition n>2).

    Réponse : D’après le GTF, on ne peut considérer que les exposants qui sont inférieurs ou égaux à deux c'est-à-dire 1 et 2 par ce que les exposants n>2, l’égalité de Fermat est impossible.

    Sommes nous d’accord ?

    Ainsi, la question qui est posée revient donc à celle de savoir si un triplet (x,y,z) d’entiers non nuls peut vérifier simultanément x+y=z et x²+y²=z².

    La réponse est évidemment non, puisque cela nous amène à une absurdité. Vous savez pourquoi ? (Evariste_galois, skydancer, rvz et Martini – répondez !)
    Tu nous prend pour des ....
    si x+y=z alors z^2 = x^2 + y^2 + 2xy

    Bon n"empeche que je n'ai pas vue de conditions du type x,y,z entiers dans ton texte et c'est ca la clé. Il existe une infinité de solutions dans les nombre réels et p-adique.

  18. #108
    invite79d10163

    Re : Nouvelle démonstration du grand théorème de Fermat

    A geatan,

    Saches que de nombreux chercheurs ont tenté de prouver la conjecture de Fermat par des voies géométriques, dont wiles qui y est parvenu avec succes. Car la voie des courbes elliptiques est sans conteste une approche géometrique du probleme.

    Dans une conférence que tu peux visionner en ligne sur CANAL U donner par Yves HELLEGOUARCH à propos du theoreme de fermat, un homme dis à la fin de la présentation, lors des questions, qu'ils existeraient une demonstration purement geometrique de la conjecture donné par Roger cotes. Donc si tu te sens le courage de la trouver, je serais interresser.... Mais je pense qu'elle doit etre inexact puisque inconnue malgré que roger cotes fut un iminent mathématicien.

  19. #109
    leg

    Re : Nouvelle démonstration du grand théorème de Fermat

    Citation Envoyé par rvz Voir le message
    Cool
    Je pensais qu'il y avait des fautes, vu que leg semblait sceptique.
    rassure toi rvz je ne suis pas sceptique , et je n'en ai pas marre de chercher loing de là

    je veux juste comprendre ce que tu fais afin de trouver la contradiction permettant de conclure que N=4, n'a pas de solution.
    de plus le lemme que j'ai posté n'est pas de moi mais d'une université et dedans ce lemme j'ai trouvé la contradiction du cas N = 4 lorsque je l'ai lu pour la premières fois
    c'est pour cela que je te l'ai posté afin de voir si tu prend le même chemin et si tu vois ce que moi j'ai trouvé!
    donc je te laisse a ton travail et j'attend de tes nouvelles bon courrage.

    gaetan:
    bien sur que je t'inviterai au resto des que ta démo et faite avec la bénédiction de Martini car moi j'ai bien peur de ne pouvoir te suivre mais d'autre en son capable alors j'attend et j'essaye de te comprendre avec les différents intervenants dommage que skydancer n'a pas démontrer le cas n=4 il a perdu son repas mais peut être qu'en ragardant le lemme 4 il trouverra une idée qui lui permettra de conclure ce cas précis
    A+

  20. #110
    invite79d10163

    Re : Nouvelle démonstration du grand théorème de Fermat

    Citation Envoyé par leg Voir le message
    dommage que skydancer n'a pas démontrer le cas n=4 il a perdu son repas mais peut être qu'en ragardant le lemme 4 il trouverra une idée qui lui permettra de conclure ce cas précis
    A+
    Je ne voulais pas démontrer ce cas. Au risque de me répeter je dis juste qu'avec l'hypothese x^2 + y^2 = z^2 on arrive à monter que x,y,z ne vérifie pas l'equation de fermat pour n>2.

    ET je suis pas à la recherche d'un repas gratuit, je pourrais t'inviter aussi, je suis sur que la discussion sera interressante

  21. #111
    invite6b1e2c2e

    Re : Nouvelle démonstration du grand théorème de Fermat

    Citation Envoyé par leg Voir le message
    je veux juste comprendre ce que tu fais afin de trouver la contradiction permettant de conclure que N=4, n'a pas de solution.
    Effectivement, plus j'y réfléchis, plus je me dis que le dernier point que j'ai écrit est faux, n'en déplaise à Zinia. J'ai tout simplement oublié de tout élever au carré.

    Rien de plus pour l'instant, désolé.

    __
    rvz

  22. #112
    martini_bird

    Re : Nouvelle démonstration du grand théorème de Fermat

    Salut,

    Citation Envoyé par Gaétan Mbama Voir le message
    3°) J’ai plutôt dis ceci : on part de ce système (page 4, Titre 3.3, formule (c)) pour aboutir au GTF (page 3, Titre 2.2, formule (b)).
    La formule (c) est fausse sur le plan logique : tu seras d'accord qu'il suffit d'un contre-exemple, or en prenant x=3, y=4, z=5, on a bien P2 () mais Qn est faux si n>2 ().

    4°) En supposant x, y et y comme mesurant les côtés d’un triangle rectangle d’hypoténuse z, j’exécute une manœuvre conceptuelle de changement de cadre mathématique qui me permet d’approcher le problème géométriquement. Cette manœuvre, en soi, n’altère pas le contenu de l’énoncé du GTF. ( Salut Zinia, tu as compris !)
    C'est cette « manœuvre conceptuelle de changement de cadre » qui est floue, pour le moins...

    5°) Tu dis que cette hypothèse est restrictive. Mais je ne vois pas quoi elle est restrictive. Je vois seulement qu’elle me permet d’écarter les solutions ‘’triviales’’ (0,0,0), (0,1,1), (1,0,1) et (1,1,0) ( comme l’a souligné à juste titre rvz).
    La condition est beaucoup plus forte que d'éliminer les solutions triviales : dans , elle élimine toutes les solutions...

    6°) Pourquoi voulez-vous que x, y et z soient forcément des nombres entiers.
    Relisons ensemble le GTF tel que traduit du latin (Martini, tu veux des références?)
    « Cubem autem in duos cubos, aut quadratoquadratum in duos quadratoquadratum, et generaliter nullam in infinitum ultra quadratum potestatem in duos ejusdem nominis fas est dividere: cuius rei demonstrationem mirabilem sane detexi. Hanc marginis exiguitas non caperet. » C'est une tarte à la crème : quand je parlais de références, c'était pour ton introduction qui fait faire des bonds à quiconque a un minimum de culture historique :

    Le premier à « tomber dans le piège » de Fermat fut Euler dont les travaux devaient définitivement orienter vers la théorie des nombres les recherches ultérieures qui allaient se faire sur cette question (ah bon ?). Cinquante plus tard, d’autres empruntèrent la même voie consacrant ainsi l’approche arithmétique (il s'agit d'un problème d'arithmétique, ça semble naturel) : Sophie Germain, Legendre, Lebesgue (que vient faire Lebesgue ici ???), que sais-je (en effet...)? Kummer (par sa théorie des nombres idéaux élaborée à d’autres fins (archi-faux : il a développé la théorie des nombres idéaux précisément pour rétablir l'unicité de la factorisation dans les extensions cyclotomiques et ainsi démontrer le GTF pour un grand nombre de cas))…Gauss s’y essaya, sans trop insister, puis désista (aucun papier n'en témoigne à ma connaissance).
    Enfin bref, la question qui nous intéresse n'est pas là de toute façon.


    Dites-moi, dans cet énoncé, où est ce qu’il est dit que x, y et z doivent être exclusivement des entiers ? Il peut aussi s’agir de rationnels!
    C'est un problème équivalent : si , avec x, y, z, a, b, c entiers, il suffit de multiplier l'équation par pour obtenir une solution entière...

    Question: en quoi, par exemple, 33 est ‘’plus cube’’ que 3,53 ?
    Par cube (ou carré, etc.), il faut entendre cube (ou carré, etc.) parfait : le cube (ou carré, etc.) d'un entier...

    Enfin, ce qui m'intéresserait le plus c'est de connaître plus en détail cette fameuse « manœuvre conceptuelle de changement de cadre ».

    Cordialement.
    « Angle éternel, la terre et le ciel, pour bissectrice, le vent. » Garcia Lorca

  23. #113
    invite79d10163

    Re : Nouvelle démonstration du grand théorème de Fermat

    Citation Envoyé par martini_bird Voir le message
    La condition est beaucoup plus forte que d'éliminer les solutions triviales : dans , elle élimine toutes les solutions...
    Je suis d'accord avec ça. De plus pour moi cette conséquence est trivial... n'est ce pas ?

    De plus le fait que Geatan doutes que x,y,z sont forcément des entiers me fait douter sur le fait qu'il est bien compris le theoreme de Fermat.

    Geatan, As-tu remarqué que l'equation de Fermat possede une infinité de solutions x,y,z dans quelque soit le nombre n ?

  24. #114
    leg

    Re : Nouvelle démonstration du grand théorème de Fermat

    Citation Envoyé par skydancer Voir le message
    De plus le fait que Geatan doutes que x,y,z sont forcément des entiers me fait douter sur le fait qu'il est bien compris le theoreme de Fermat.
    il faut quand même pousser,
    car effectivement, il n'est pas obliger d'utiliser des entiers
    pour vérifier l'équation de Fermat heureusement d'ailleur,
    car la démo de A Wiles serait fausse.

    pour moi
    x,y et z dans mon travail représente 3 racines carrées
    donc pour N>2 impair et N premier x, y et z ne sont pas des entiers ex: racine carrée de 23, 3^3,5^3, 6^3 ...n^3 avec n entier naturel non carré et alors est ce que cela est une contradiction ou une impossibilite de démontrer Fermat pour N = 3?

    par contre il est évident qu'en utilisant cette méthode dans les puissances N pairs, x, y et z sont des racines carrées entieres, donc des entiers mais qui ne pourrait résoudre qu'une partie des entiers élevés à la puissance N=3 donc se serait réstrictif et ne pourrait résoudres toutes les équation de Fermat pour N=3.

    pour t'en convaincre regarde un peu le lemme que j'ai posté et dit moi si tu ne remarque rien de curieux
    avec les triplets pythagoriciens, tu vois je te donne une indication..

  25. #115
    invite7863222222222
    Invité

    Re : Nouvelle démonstration du grand théorème de Fermat

    Gaetan,

    Zinia a bien montré que ca ne réduit en rien la portée de ta démonstration que de considérer un triangle quelconque pas seulement rectangle.

    Ainsi, surement que ta démonstration doit pouvoir tenir en quelques pages si tu considères seulement n > 2 puisque c'est ce qui est supposé dans GTF.

    Je ne dis pas que les autres cas ne sont pas interessants ou plutot "lié" dans le cadre de ta démonstration mais je dis que c'est mieux de partir du plus général pour après dire que c'est aussi applicable pour des valeurs de n données (1 et 2).

    Si j'ai bien compris, tu devrais seulement différencier deux cas : l'angle est obtu ou pas.

    De plus, personellement, j'ai eu un peu de mal à comprendre comment tu détermines les intervalles de n pour lequel on a f1(n) = f2(n) quelque soit x, y et z.

    Mais déjà si tu arrives a démontré que necessairement les valeurs n sont comprise (je dis n'importe quoi) entre 3 et 49, alors c'est déjà un résultat important.

    Quand aux valeurs possibles de x, y et z et au fait qu'ils ne peuvent pas être entiers, j'ai pas l'impression non plus avoir lu une explication.

  26. #116
    invite79d10163

    Re : Nouvelle démonstration du grand théorème de Fermat

    Citation Envoyé par leg Voir le message
    il faut quand même pousser,
    car effectivement, il n'est pas obliger d'utiliser des entiers
    pour vérifier l'équation de Fermat heureusement d'ailleur,
    car la démo de A Wiles serait fausse.
    Qu'est ce que tu entends par vérifier ? Je vois pas ce que tu veux dire.

    Le fait que l'equation de fermat soit vérifié par une infinité de nombre réels n'est pas un fait nouveau.
    Le fait qu'aucune solution ne soit un triplet d'entiers constitue le theoreme de Fermat-Wiles.

    Si Geatan veut demontrer Fermat il doit s'attaquer au nombre entiers, ça c'est sur. Ou bien il peut essayer de démontrer des conjectures liés au théoreme de Fermat, mais là c'est une autre histoire.

    Pour le reste je vais y réflechir plus tard...

  27. #117
    Quinto

    Re : Nouvelle démonstration du grand théorème de Fermat

    Citation Envoyé par leg Voir le message
    il n'est pas obliger d'utiliser des entiers pour vérifier l'équation de Fermat heureusement d'ailleur, car la démo de A Wiles serait fausse.
    Bein voyons...
    Si les nombres ne sont pas entiers, c'est trivial de voir qu'il existe une infinité de solutions.

  28. #118
    invite636fa06b

    Re : Nouvelle démonstration du grand théorème de Fermat

    Citation Envoyé par rvz Voir le message
    Effectivement, plus j'y réfléchis, plus je me dis que le dernier point que j'ai écrit est faux, n'en déplaise à Zinia. J'ai tout simplement oublié de tout élever au carré.
    rvz
    Mille excuses !
    Comme punition j'indique la suite de la démo :
    il faut continuer avec le constat que x²=n²-p² donc qu'ils constituent un triplet pythagoricien et qu'il existe c et d tels que p=2cd, x=c²-d², n=c²+d²
    donc y²=2np=4cd(c²+d²).
    On montre de manière identique que c,d, et c²+d² sont premiers entre eux et donc qu'ils sont chacun un carré.
    Il existe donc e,f et g tels que c=e²,d=f²,c²+d²=g²
    Soit encore g²=e^4+f^4
    Ainsi partant d'un triplet Z²=X^4+Y^4 on démontre l'existence d'un autre triplet strictement plus petit vérifiant la même équation. Par cette descente infinie on montre que l'équation en Z² n'a pas de solution et a fortiori celle en Z^4 non plus.
    NB on ne peut pas s'en sortir avec les modulos

    Enfin si je ne me suis pas encore fourvoyé...

  29. #119
    leg

    Re : Nouvelle démonstration du grand théorème de Fermat

    Citation Envoyé par Gaétan Mbama Voir le message
    Leg salut !


    Par contre, quand j’écris par exemple l’égalité x1,15+y1,15=z1,15 , pour moi, elle a un sens géométrique précis c'est-à-dire x, y et z représentent les côtés d’un triangle obtus dont je peux, avec précision, déterminer l’angle. Donc, je peux passer d’un exposant n à un autre par une simple rotation. Ceci explique pourquoi un même triplet ne peut pas vérifier l’égalité de Fermat dans deux exposant différents.
    La méthode permet d’établir un lien entre l’exposant n de l’égalité de Fermat xn+yn=zn et l’angle entre les cathètes du triangle. (La formule n’est pas dans le texte).

    Question ?

    Amicalement !
    a)
    si un même triplet ne peut vérifier l'équation de Fermat dans deux exposants différent d'accord;
    mais est ce pour autant que cela vérifie par exemple toutes les équations de Fermat pour l'un ou l'autre des exposants

    b)peut être qu'il te faut mettre la formule, afin de vérifier qu'effectivement cela représente bien l'égalité de Fermat quelque soit l'exposant premiers >2 et c'est peut être sur ce point qu'il y aurra discution

    j'espère que tu n'as pas trop fain car il va te falloir attendre A+

  30. #120
    invite6b1e2c2e

    Re : Nouvelle démonstration du grand théorème de Fermat

    Citation Envoyé par zinia Voir le message
    Mille excuses !
    Comme punition j'indique la suite de la démo :
    héhé ! D'accord, c'est un peu moi qui t'ai induit en erreur

    En tout cas, j'adore ta manière de te faire pardonner. Et cette fois ci je confirme pour leg que c'est juste.

    Merci aussi de m'avoir remis la démo en tête.

    __
    rvz

Page 4 sur 8 PremièrePremière 4 DernièreDernière

Discussions similaires

  1. Une démonstration analytique du théorème de Fermat
    Par invitede6b99d6 dans le forum Mathématiques du supérieur
    Réponses: 16
    Dernier message: 29/09/2011, 13h15
  2. Généralisation du grand théorème de Fermat
    Par breukin dans le forum Mathématiques du supérieur
    Réponses: 15
    Dernier message: 02/09/2006, 19h31
  3. Réponses: 7
    Dernier message: 21/06/2005, 02h26
  4. Le dernier théorème de Fermat.
    Par Antikhippe dans le forum Lectures scientifiques
    Réponses: 41
    Dernier message: 02/09/2004, 18h43